When is a Particle at Rest in a Motion Problem?

  • Thread starter Thread starter Sage Hopkins
  • Start date Start date
  • Tags Tags
    Particle Rest
Click For Summary
A particle is at rest when its velocity, v(t), equals zero, which leads to the equation v(t) = - (π/4) sin (πt/4) = 0. This simplifies to sin(πt/4) = 0, resulting in the solutions t = 0, 4, and 8 seconds, derived from the general solution πt/4 = πn. The full problem involves determining when a particle described by the motion law s = cos(πt/4) is at rest for t ≤ 10 seconds. Understanding this requires knowledge of the unit circle and the sine function, which clarifies the relationship between the sine values and the times when the particle is at rest. The discussion emphasizes the importance of differentiating the motion function and reviewing trigonometric concepts.
Sage Hopkins
Messages
2
Reaction score
0
(1.)

I have a "particle in motion" problem that is asking me when a particle is at rest, which I understand to be when velocity = v(t) = 0, so

v(t) = - (π/4) sin (πt/4) = 0.

The given answer is as follows:

- (π/4) sin (πt/4) = 0

sin (πt/4) = 0

πt/4 = πn.

t = 0,4,8 seconds.

(2.) Can someone please explain to me how 0 becomes πn, and/or what specific mathematical concept(s) I need to review?
 
Physics news on Phys.org
What's the full problem statement?

Of course \sin(n \pi)=0 for all n \in \mathbb{Z}.
 
The full problem statement goes:

A particle moves according to a law of motion s = cos(πt/4), t >= 0, where t is measured in seconds and s in feet.

There are several sub-questions from here about velocity, acceleration, graphs, etc.., but the one that I got stuck on is

(c.) When is the particle at rest for t <= 10.

After differentiating the given function for f', understanding the answer to this question was as simple as reviewing the unit circle and the graph of sin for me, as elementary as it may be.

Thank you for your time!
 
Question: A clock's minute hand has length 4 and its hour hand has length 3. What is the distance between the tips at the moment when it is increasing most rapidly?(Putnam Exam Question) Answer: Making assumption that both the hands moves at constant angular velocities, the answer is ## \sqrt{7} .## But don't you think this assumption is somewhat doubtful and wrong?

Similar threads

  • · Replies 1 ·
Replies
1
Views
2K
  • · Replies 20 ·
Replies
20
Views
3K
  • · Replies 5 ·
Replies
5
Views
2K
  • · Replies 4 ·
Replies
4
Views
2K
  • · Replies 1 ·
Replies
1
Views
9K
Replies
2
Views
4K
  • · Replies 10 ·
Replies
10
Views
2K
  • · Replies 4 ·
Replies
4
Views
2K
Replies
8
Views
1K
  • · Replies 7 ·
Replies
7
Views
5K